Вы находитесь на странице: 1из 102

Team of Microeconomics I Teaching Assistant | 1

INTRODUCTION TO MICROECONOMICS
MODULE

TEACHING ASSISTANTS OF MICROECONOMICS AND


MACROECONOMICS
UNDERGRADUATE ECONOMICS PROGRAM
FACULTY OF ECONOMICS AND BUSINESS
UNIVERSITAS PADJADJARAN
2018

Team of Microeconomics I Teaching Assistant | 2


ACKNOWLEDGEMENT
In the name of Allah, The Most Gracious, The Most Merciful

Alhamdulillah, all praises to Allah SWT, The Almighty, for giving belief,
health, confidence and blessing for the writers to accomplish this Module
of Introduction to Microeconomics. Shalawat and Salam be upon our Prophet
Muhammad SAW, who has brought us from the darkness into the brightness
and guided us into the right way of life.
In this opportunity, we also like to express our deep thanks to Head
Department of Economics, Coordinator of Undergraduate Program of Department of
Economics, lecturers, and those who contributed and helped in the process of making
this module. All of your kindness and help means a lot to us. Thank you very much
We realise that the contents in this module is not that perfect. Therefore, we are
willing to receive and consider feedback, suggestions and constructive criticisms, and
eager to implement improvements.
Hopefully this module can be the short guide for the students
in order to deepen their understanding and analysis of Macroeconmics theory.
Thank you.

List of the Module Writers:

1. Putri Riswani Halim 120210150086


2. Irmayanti 120210150014
3. Ade Faisal Fahroez 120210150005
4. Ibnu Ahmadsyah 120210140106
5. Farah Muthia Syifa 120210150087
6. Gianti Haidi Mazaya 120210140061
7. Nissa Alifia Elista 120210150115

Acknowledge and Agree,


Coordinator of Undergraduate Program of Department of Economics

Rudi Kurniawan, S.E., MSi., Ph.D.


NIP. 197003101997021002

Team of Microeconomics I Teaching Assistant | 3


MODULE AND TUTORIAL GUIDELINES

1. This module was created as a media to help the students deepen their
understanding during the tutorial sessions of Introduction to
Microeconomics.

2. All of the problems in this module are provided in English.


3. The types of the problems are Fill in the blank, True or False, Multiple
Choice, and Essay.

4. This module could only be used during the tutorial sessions of


Macroeconomics.

5. The students are not allowed to bring and copy the module unless they
obtained permissions from the Team of Teaching Assistant.

6. For any reasons, the students are not allowed to write anything in the module
unless they obtained permissions from the Team of Teaching Assistant.

7. The answers are written on the answer sheet/other paper that has been
provided by the Team of Teaching Assistant.

8. The materials in each tutorial meeting are adjusted based on the materials
that have been given by each of the lecturers in the class.

9. During the tutorial, all of the students should obey the rules that have been
made by each of the Teaching Assistant.

10. The maximum duration for laboratory is 2.5 hours (180 minutes)
11. For any incorrect or unclear questions that you found difficult, please re-read
the appropriate question or ask directly to the Teaching Assistant to clear up
any confusion.

Team of Microeconomics I Teaching Assistant | 4


12. After successfully finishing the problems, the students can leave the
tutorial room with the permission from the Teaching Assistant.

13. Here below we kindly inform the general rule during the laboratory:
• The tutorial has 10 (ten) meetings.
• The students are not allowed to change their tutorial schedule without
any permission from their Teaching Assistant.

• Other rules could be made later on by the Teaching Assistant with the
consent of the students in each tutorial.

Team of Microeconomics I Teaching Assistant | 5


CHAPTER 1
THE SCOPE AND METHOD OF ECONOMICS & THE
ECONOMIC PROBLEM: SCARCITY AND CHOICE

• Economic is the study of how individual (a person, a firm, or even a


country) allocate the scarce resources in order to optimize his/her
objectives.

• Economics ask and attempt to answer two kinds of questions: positive and
normative. Positive economics attempt to understand behavior and the
operation of economies without making judgments about whether the
outcomes are good or bad. Normative economics look at the results of
economic behavior and asks whether the results are good or bad.

• Models and theories can be expressed in many ways. The most common
ways are in words, in graphs, and in equations.

• Empirical economics involve the collection and use of data to test economic
theories. In principle, the best model is the one that yields the most accurate
prediction.

• Because resources are scarce relative to human wants in all societies, using
resources to produce one good or service implies not using them to produce
something else. This concept of opportunity cost is central to an
understanding of economics.

• In some modern societies, government plays a big role in answering the


three basic questions. There are neither purely planned economics and nor
purely laissez-faire economics; all economics are mixed.

Team of Microeconomics I Teaching Assistant | 6


CHAPTER 1
THE SCOPE AND METHOD OF ECONOMICS & THE
ECONOMIC PROBLEM: SCARCITY AND CHOICE

Fill in the Blank

1. ________ is concerned with household income, ________deals with


national income.

2. The alternative that we give up when we make a choice or a decision is


called _______.

3. Cost that cannot be avoided because they have already been incurred is
called ________.

4. If you decide to take time off from work, the opportunity cost of your leisure
is________.

5. A device used to analyze the relationship between two variables while the
values of other variables are held unchanged is called ________.

6. Criteria for judging economic outcomes are ________, ________,


________, and ________.

7. The inputs into the process of production is called ________.


8. ________ is the slope of PPF.
9. Economic growth will shift PPF curve to the ________ & ________.
10. A producer has a/an ________ over another in the production of a good or
service if it can produce that product using fewer resources.

Team of Microeconomics I Teaching Assistant | 7


True or False

1. Economic is the study of how individuals and societies choose to use the
scarce resources that nature and previous generations have provided.

2. Opportunity costs arise because resources are scarce.


3. Normative economics describes what exists and how it works.
4. Economic theory is a statement or set of related statements about cause and
effect, action and reaction.

5. The compilation of data that describe phenomena and facts is called


empirical economics.

6. Human wants are limited but resources are unlimited.


7. Buildings, equipment, desks, chairs, software, roads, bridges, and highways
are a part of the nation’s stock of capital.

8. An efficient economy is one that produces what people want at the least
possible cost.

9. Producers are those who take resources and transform them into usable
products, or outputs.

10. Improved productivity may come from additional capital or from the
discovery and application of new, more efficient techniques of production.

Team of Microeconomics I Teaching Assistant | 8


Multiple Choice

1. An approach to economics that seeks to understand behavior and the


operation of systems without making judgements is called …

a. normative economics
b. positive economics
c. descriptive economics
d. empirical economics

2. A formal statement of a theory, usually a mathematical statement of a


presumed relationship between two or more variables is ….

a. Model
b. Graphic
c. Diagram
d. Formula

3. These are four specific criteria in judging the economic performance,


except ….

a. Efficiency
b. Equity
c. Growth
d. Wealth

4. Which one of these statements below that’s not explaining an empirical


economics?

Team of Microeconomics I Teaching Assistant | 9


a. Economists studying the labor market can now test behavioral
theories against the actual working experiences of thousands of randomly
selected people who have been surveyed continuously since the 1960s.
b. Monitoring and studying the behavior of the national economy at the
National Bureau of Economic Research (NBER) pass thousands of
items of data, collected by both government agencies and private
companies

c. Keith Chen of Yale, has used experiments with monkeys to investigate


the deeper biological roots of human decision making.

d. Government should reconsider about the rise of fuel price.

5. An increase in the total output of an economy is called ….


a. Inflation
b. Saving
c. Growth
d. Rise

6. These are the reasons to study economics, except ….


a. to learn a way of thinking
b. to understand society
c. to understand global affairs
d. to rule the world.

7. These are the basic questions that must be answered to understand the
functioning of the economic system, except ….

a. What gets produced?


b. How is it produced?

Team of Microeconomics I Teaching Assistant | 10


c. Who gets what is produced?
d. How much is the cost of production?

8. These are economic system that usually used by countries, except ….


a. Commando
b. Market
c. Laissez-faire
d. Mixed

9. Factor that will move PPF curve to the left is ….


a. Expansion
b. Growth
c. Good technology invention
d. Recession

10. The PPF illustrates a number of important economic concepts, except….

a. scarcity
b. unemployment
c. inefficiency
d. population

Essay

1. There are four criteria for judging economic outcomes, what are they?
Explain each criteria!

2. Explain briefly the differences between macroeconomics and


microeconomics!

Team of Microeconomics I Teaching Assistant | 11


3. What is efficient market? What is the meaning of “no free lunch” related to
it?

4. Economics asks and attempts to answer two kinds of questions:


positive and normative. Explain and give an example of each!
5. Explain briefly the concept of ceteris paribus!
6. For each of the following, describe some of the potential opportunity costs:

a. Studying for your economics test


b. Spending 2 hours playing computer games
c. Going to graduate school
d. Buying a new car instead of keeping the old one
7. What is the relationship between scarcity, choices, and opportunity cost?

8. Explain the difference between comparative and absolute advantage using


an example and draw the PPF!

9. Draw PPF curve and explain briefly all factors that shift PPF curve!
10. “As long as all resources are fully employed, and every firm in the economy
is producing its output using the best available technology, the result will
be efficient.” Do you agree or disagree with the statement? Explain your
answer.

Team of Microeconomics I Teaching Assistant | 12


CHAPTER 2
DEMAND, SUPPLY, MARKET EQUILIBRIUM & ITS
ELASTICITY

• The law of demand indicates a negative relationship between price level


and quantity of demand in a period so when price level increased causes
quantity of demand decreased, vice versa.

• Quantity demanded may change due to changes in the price level that
caused movement along the demand curve, while the demand for most
goods may change due to changes in income, wealth, tastes, prices of
other goods, and expectations that caused a shift in the demand curve.

• The law of supply indicates a positive relationship between price level


and quantity of supply in a period so when price level increased causes
quantity of supply increased, vice versa.

• Quantity supplied may change due to changes in the price level that
causes movement along the supply curve, while the supply for most
goods may change due to changes in cost of production, and prices of
related goods that caused a shift in the supply curve.

• Market equilibrium occurs when the quantity demanded equals the


quantity supplied which resulting equilibrium price level and
equilibrium output. When the quantity demanded exceeds the quantity
supplied will occur excess demand (shortage), could be caused by a
price ceiling which the price level is below the equilibrium price. When
the quantity supplied exceeds the quantity demanded will occur excess
supply (surplus), could be caused by a price floor which the price level
is above the equilibrium price.

Team of Microeconomics I Teaching Assistant | 13


• Resource allocation is the output of every economic system! It does
not exclusively belong market system, price rationing is the market
system distributes goods and services on the basis of willingness and
ability to pay.

• Consumer surplus is the difference between the maximum amount a


person is willing to pay for a good and its current market price, the area
is below a demand curve and above a price level. Producer surplus is
difference between the full cost of production for the firm and the
current market price, the area is above a supply curve and below a price
level.

• The dead weight loss is the net loss in consumer surplus and producer
surplus due to reduced production or excess in production.

• Elasticity is a general concept that can used to quantify the response in


one variable when another variable changes. Price elasticity of demand
measures how responsive consumers are to changes in the price of a
product.

Price elasticity of demand =

• The types of elasticity of demand are perfectly inelastic (Ed=0), inelastic


(0<Ed<1), unitary elastic (Ed=1), elastic (Ed>1), and perfectly elastic
(Ed=~).

• The cross elasticity = ; when


E>0
indicates both of goods are substitutes and when E<0 indicates both of
goods are complements.

Team of Microeconomics I Teaching Assistant | 14


CHAPTER 2
DEMAND, SUPPLY, MARKET EQUILIBRIUM & ITS
ELASTICITY

Fill in the Blank

1. The law of demand states that a _________ price leads to a _________ quantity
demanded and that _________ price leads to a _________ quantity demanded.
2. Demand curve has ________ slope indicates the relationship between ________
and ________ 

3. A lower price for a substitute ________ demand for the other product.
4. A technological improvement that ________ costs of production will shift supply
to the ________.
5. The Elasticity of demand is unitary inelastic when _______.
6. Quantity of demand not respond at all when there are changes in 
prices, the
kind of elasticity of demand is ________.
7. The net loss in consumer surplus and producer surplus due to reduced production
or excess in production is called _______.
8. ________ is when the quantity demanded exceeds the quantity supplied will
occur excess demand 

9. ________ which the price level is above the equilibrium price. 

10. The elasticity of demand when corn sales increased 50% when the 
price
decreased 25% is ________.

Team of Microeconomics I Teaching Assistant | 15


True or False

1. An increase in the supply of lettuce decreases its equilibrium price and


increases its equilibrium quantity

2. A price floor set by government will increase the equilibrium price and
quantity in a Market

3. Assume that a downsloping demand curve Intersects a vertical supply


curve. A leftward shift in the supply curve will result in a surplus of a
product

4. An improvement in production technology will shift the supply curve to


the left. 


5. The supply function is Qs=85+0,5Px, when the price is $100, then


quantity supplied of X is 350 units. 


6. In the market for sushi, an equal increase in supply and demand will
increase the equilibrium price, but have no effect on the equilibrium
quantity

7. Increasing in the apple price by 40% led to an decrease in the quantity of


apple demanded by 60%, then the elasticity of demand is an inelastic type.

8. In the market for gasoline, the reason that equilibrium quantity increased
was that the increase in demand was less than the decrease in supply

9. In the input market, households offer factors of production while the firms
made the demand factors of production. 


10. Percentage an increase in the price of good Y is 40% causes a decrease in


the quantity demanded of the X by 20%, then good X and good Y are
complements.

Team of Microeconomics I Teaching Assistant | 16


Multiple Choice

1.If an increase in the price of paper leads to a decrease in the demand for
pencils, then paper and pencils are:

a. substitutes

b. complements

c. normal goods

d. inferior goods

2.Thelaw of demand states that an increase in the price of a good:

a. decreases the demand for that good

b. increases the supply of that good

c. decreases the quantity demanded for that good

d. increases the quantity supplied of that good

3.The law of supply states that an increase in the price of a good:

a. decreases the demand for that good

b. increases the supply of that good

c. decreases the quantity demanded for that good

d. increases the quantity supplied of that good

Team of Microeconomics I Teaching Assistant | 17


4. If an increase in consumer income leads to a decrease in the demand for good
x, then

good x is:

a. a substitute

b. a complementary good

c. a normal good

d. a inferior good

5. Which of the following shifts the demand for watches to the right?

a. a decrease in the price of watches

b. a decrease in consumer income if watches is a normal good

c. an increase in the price of watches

d. an increase in consumer income if watches is a normal good

6. All of the following shift the supply curve of watches to the right except:

a. an advance in the technology used in manufacturing watches

b. a decrease in the wages/salaries of workers employed to manufacture


watches

c. an increase in the price of watches

d. manufacturers’ expectations of lower watch prices in the future

Team of Microeconomics I Teaching Assistant | 18


7. If the price of a good is above the equilibrium price,

a. there is a surplus and the price will fall

b. there is a surplus and the price will rise

c. there is a shortage and the price will fall

d. there is a shortage and the price will rise

8. If price elasticity of demand is -0.5.

a. a 1% decrease in quantity demanded leads to a 0.5% decreae in 
price. 


b. a 1% decrease in price leads to a 0.5% increase in quantity 
demanded. 


c. a 50% decrease in price leads to a 1% increase in quantity 
demanded. 


d. Demand is elastic. 


9.Producer surplus in the market is ...

a. Above the supply curve and above the price level 


b. Below the demand curve and above the price level 


c. Below the demand curve and below the price level 


d. Above the supply curve and below the price level

Team of Microeconomics I Teaching Assistant | 19


10. When demand is price inelastic, total revenue is

a. Directly related to quantity demanded 


b. Inversely related to quantity demanded 


c. Directly related to price 


d. Not related to either price or quantity demanded

Essay

1. Give complete explanation about law of supply and law of demand! And
complete your answer with the graph!

2. Using supply and demand diagrams, show the effect of the following
events on the market for sweatshirts.

a. A drought in Egypt damages the cotton crop


b. The price of leather jackets falls.
c. All universities require students to attend morning exercise classes in
appropriate attire.
d. New knitting machines are invented.

3. Suppose that the price of tickets to your local football team play at home
are determined by market forces. Currently, the supply and demand
schedules are given below.

P Q Demanded Q Supplied
10 50,000 30,000
20 40,000 30,000
30 30,000 30,000
40 20,000 30,000
50 10,000 30,000

Team of Microeconomics I Teaching Assistant | 20


a. Draw the demand and supply curves. What is unusual about this
supply curve? Why might this be true?
b. What are the equilibrium price and quantity of tickets?

4. Market research has revealed the following information about the market
for chocolate bars: the demand schedule can be represented by the
equation QD=1600-300P where QD is quantity demanded and P is price.
The supply schedule can be represented by the equation QS=1400 +700P
where QS is the quantity supplied. Calculate the equilibrium price and
quantity in the market for chocolate bars
5. In December, the price of Christmas trees rises and the quantity of trees
sold also rises. Is this a violation of the law of demand?
6. A firm charges $800 for its unique word processor. If total revenue is
$56,000 in July, how many word processors were sold that month?
7. Which of the following factors will cause the demand curve for labor to
shift to the right?
(a) the demand for the product by labor declines.
(b) the prices of substitute inputs fall.
(c) the productivity of labor increases.
(d) the wage rate declines
(e) None of the above.

8. Demand and supply in a market are described by the equations


Qd = 66-3P
Qs = -4+2P
a. Solve algebraically to find equilibrium P and Q
b. What is the equilibrium P and Q if the per unit tax is t=5
c. Illustrate the pre-tax equilibrium and the post-tax equilibrium on a
graph

Team of Microeconomics I Teaching Assistant | 21


9. Houses on the Bosphorus have an inelastic supply. Cars have an elastic
supply. Suppose that growth in the population of Istanbul doubles demand
(at every price) for both products. Delete the incorrect answer:

 The equilibrium price of cars will rise/fall.


 The equilibrium price of houses will rise/fall.
 The equilibrium quantity of cars will rise/fall.
 The equilibrium quantity of houses will rise/fall.
 Houses/cars will have the larger change in price.
 Houses/cars will have the larger change in quantity.
 Total consumer spending on cars will rise/fall.
 Total consumer spending on houses will rise/fall.

10. Give a complete explanation and example of each about types of elasticity
of demand!

Team of Microeconomics I Teaching Assistant | 22


CHAPTER 3
HOUSEHOLD BEHAVIOUR & CONSUMER’S CHOICE

• It is best to think of the household choice problem as one of allocating


income over a large number of goods and services. A change in the price
of one good may change the entire allocation. Demand for some goods
may rise, while demand for others may fall.

• Households allocate income among goods and services to maximize


utility. This implies choosing activities that yield the highest marginal
utility per dollar. In a two-good world, households will choose to equate
the marginal utility per dollar spent on X with the marginal utility per
dollar spent on Y.

• In the labor market, a trade-off exists between the value of the goods
and services that can be bought in the market or produced at home and
the value that one places on leisure. The opportunity cost of paid work
is leisure and unpaid work. The wage rate is the price, or opportunity
cost, of the benefits of unpaid work or leisure

• In addition to deciding how to allocate its present income among goods


and services, a household may also decide to save or borrow. When a
household decides to save part of its current income, it is using current
income to finance future spending. When a household borrows, it
finances current purchases with future income.

Team of Microeconomics I Teaching Assistant | 23


• Optimality Condition

Team of Microeconomics I Teaching Assistant | 24


CHAPTER 3
HOUSEHOLD BEHAVIOUR & CONSUMER’S CHOICE

Fill in the Blank

1. ___________is information on household income, wealth, and prices that


tells us what choices are possible for households.
2. Within the constraints, households are free to choose what they will buy
based on __________
3. When the price of good X and Y increase, budget constraint will
__________
4. ___________occurs when a price of good X rise then consumers will
change their consumption to good Y.
5. When the price of something falls, ceteris paribus, we are better off and
likely to buy more of that good and other goods. It is called _________.
6. Law of diminishing marginal utility stated that the ____________ of any
one goods consumed, the __________ satisfaction (utility) generated by
consuming each additional (marginal) unit of that good.
7. The ratio at which a household is willing to substitution good Y for good X
is called _________
8. Any movement away from origin point in indifference curves means that
consumer’s utility has __________ .
9. Utility maximization will take place at that point at which
____________ is just tangent to ________
10. Products that are identical to or indistinguishable from one another is
_________.

True of False

1. Financial capital market is the complex set of institutions in which suppliers


of capital (firms that save) and the demand for capital (household wanting
to invest) interact.

Team of Microeconomics I Teaching Assistant | 25


2. Negative relationship between price and quantity demanded lies in income
effect and regulatory effect.
3. Yuda is thirsty and he wants to buy some drinks. Yuda really likes to drink
Thai tea. Dumdum and Daun Muda are both a Thai tea company. Dumdum
cost Rp20,000 and offer a 40% off, while Daun Muda cost Rp10,000. Yuda
will be better off if he buy Thai tea from Dumdum.
4. Substitution effect is when things with the greatest value in use frequently
have no value in exchange and the things with the greatest value in exchange
frequently have little or no value in use.
5. Buying more units of X increases the marginal utility derived from
consuming additional units and decreases total utility of good X.
6. The budget constraint comes from price, income, and opportunity cost.
7. Indifference curve shows a combination of consumer’s preference and their
limitation on buying several goods.
8. There’s nothing in this world that cannot be measured, including utility. It
can be measured by unit of money.
9. Valli is a shoes collector. When the shoes price falls, Valli start to increase
his consumption. Now Valli is feeling richer than before. Thus, it is describe
an income effect.
10. As the wage increases, a worker will substitute income for leisure time.

Multiple Choice

1. Suppose that the income of the consumer equals $20. Use the information
on the graph to determine the prices of goods Y and X.

Team of Microeconomics I Teaching Assistant | 26


a. PY = $2, PX = $4.
b. PY = $2, PX = $3.
c. PY=$2, PX = $6.
d. PY = $20, PX = $12.

2. Ann is a student with a fixed budget income of $150. Assumed that she spend her
income on Korean foods and going to cinemas. Korean foods cost $30 and movie
ticket cost 50% cheaper than Korean foods. How many meals and movie ticket
can Ann afford?
a. Kfoods = 30; Cinemas = 15
b. Kfoods = 15; Cinemas = 30
c. Kfoods = 5; Cinemas = 10
d. Kfoods = 10; Cinemas = 5

3. Which one of the following statements are true?


a. Total utility decreases when marginal utility decreases
b. If we impose a constraint of limited income and fixed prices on households,
those households would not be free to choose which goods to buy and not
to buy

Team of Microeconomics I Teaching Assistant | 27


c. The more of any one good consumed in a given period, the less satisfaction
(utility) generated by consuming each additional unit of the same good
d. Price of leisure is a marginal utility you get from working

4. In financial capital market, firm is ____________ and household is ___________


a. borrower ; saver
b. saver ; borrower
c. both plays the same function
d. none of the above

5. Putri has a fixed budget for two goods, basketball and drawing book. Assumed
that drawing book cost $5 and basketball cost two times higher than drawing
book. Marginal utility of basketball and drawing book each are 60 and 15. Should
the consumption of basketball or drawing book be higher, lower, or remain the
same?
a. Both the consumption of basketball and drawing book should increase
b. Consumption of basketball should increase and consumption of drawing
book should decrease
c. The current combination of goods maximizes total utility. Consumption
should remain the same.
d. Both the consumption basketball and drawing book should decrease

6. Every household must make three basic decisions, except


a. how much of each product to demand
b. how much to save for the future
c. how much exchange rate will fluctuate
d. how much labor to supply

7. The following statements are the characteristics of perfect competition market.


Which one is true?
a. Homogeneous products and perfect information
b. Identical products and imperfect information
c. Differentiated products and asymmetric information
d. Homogenous information and imperfect products

8. An increase in interest rates has a positive effect on saving if ___________


a. income effect dominates the substitution effect
b. substitution effect dominates the income effect
c. both income and substitution have the same effect

Team of Microeconomics I Teaching Assistant | 28


d. none of the above

9. What is the utility-maximizing rule?


a. MUx/Py = MUy/Px
b. MUx/MUy = Px/Py
c. MUx/Px = MUy/Py
d. MUx/Px

10. Which two points yield the same total utility?

a. Point w and z
b. Point z and e
c. Point b and z
d. Point b and e

Essay

1. For this problem, assume that Kendrick has $144 to spend on cigars and
brandy each month and that both goods must be purchased whole (no
fractional units). Cigars cost $6 each month and brandy costs $30 per bottle.
Kendrick’s preferences for cigars and brandy are summarized by the
following information:

Team of Microeconomics I Teaching Assistant | 29


a. Fill in the figures for marginal utility and marginal utility per dollar for both
cigars and brandy.
b. Are these preferences consistent with the law of diminishing marginal
utility? Explain briefly.
c. Given the budget of $144, what quantity of cigars and what quantity of
brandy will maximize Kendrick’s level of satisfaction? Explain briefly.
d. Now suppose the price cigars rises to $8. Which of the columns in the table
must be recalculated? Do the required recalculations.

2. Explain why in product markets the substitution and income effects work in the same
direction for normal goods, but in the labor market, the income and substitution
effects work in opposite directions when leisure is considered normal good.

3. Suppose the price of X is $5 and the price of Y is $10 and a hypothetical household
has $500 to spend per month on goods X and Y.
a. Sketch the household budget constraint.
b. Assume that the household splits its income equally between X and Y.
Show where the household ends up on the budget constraint.
c. Suppose the household income doubles to $1,000. Sketch the new budget
constraint facing the household.
d. Suppose after the change the household spends $200 on Y and $800 on X.
Does this impy that X is a normal or an inferior good? What about Y?

4. Farah is currently consuming 3 cups of tea and 5 fried bananas every morning for
breakfast at her local cafeteria. At Rp4,500/cup, tea is twice more expensive than a
fried banana. However, at the current consumption combination Farah is willing to
trade a fried banana for a cup of tea and she will feel equally satisfied. Should she

Team of Microeconomics I Teaching Assistant | 30


decline the third cup of tea and get more bananas instead to increase her satisfaction?
Please explain.

5. Given: budget = $4,000; Pa = $50; Pb = $40 (“a” and “b” are goods). For all the
problems below, calculate the relevant numbers for the graph.
a. Draw the budget line (L1)
b. What happen to L1 if Pb is raised by 25%? Show the new budget line (L2)
on your graph.
c. What happen to L1 if Pa is cut by 50%? Draw the new budget line (L3)
d. What happen to L1 if budget is halved? Draw the new budget line (L4)

6. Irma is replacing her old refrigerator, either with something larger (volume) or more
efficient (less energy consumption). A larger capacity fridge goes for $20 more for
every litre of additional internal volume, while every 10 watts less energy
consumption raises the price by $10. After comparing her old refrigerator against the
newer models, both the larger and the more efficient ones, she decides that she is
currently willing (indifference) to trade-off 30 watts of less energy for 2 litre of more
space. Assuming that Irma is a “homo economicus”, which choice will increase her
utility: a larger fridge or a more efficient one (less energy consumption)?

7. Adrian has $21 to spend on energy drinks and protein bars and wants to maximize
his utility on his purchase. Based on the data in the table, how many energy drinks
and protein bars should Adrian purchase, and what is his total utility from the
purchase? Does the utility-maximizing rule hold true for his purchase? Explain.

8. For most normal goods, the income effect and the substitution effect work in the
same direction; so when the price of good falls, both the income and substitution
effects lead to a higher quantity demanded. How would this change if the good is an
inferior good?

Team of Microeconomics I Teaching Assistant | 31


9. The average cost of living is approximately the same in the following three cities in
the United Kingdom: Nottingham, Manchester and Bristol. Use the information you
have learned about marginal utility and the substitution effect to explain whether you
believe your purchasing choices would remain the same in each of these cities.
Assume that your income would be the same in each city.

10. What’s the difference between opportunity cost and trade off?

Team of Microeconomics I Teaching Assistant | 32


CHAPTER 4
THE PRODUCTION PROCESS

 Production is the process by which inputs are combined, transformed, and


turned into outputs.
 All firms must make several basic decisions to achieve maximum profits: (1)
How much output to supply, (2) Which production technology to use, (3)
How much of input to demand.
 Profit (economic profit) is the difference between total revenue and total
cost. Profit = total revenue - total cost
 Total revenue is the amount received from the sale of the product (q x P).
 Total cost (total economic cost) is the total of (1) out-of-pocket costs, (2)
normal rate of return on capital, and (3) opportunity cost of each factor of
production.
 Normal rate of return is a rate of return on capital that is just sufficient to
keep owners and investors satisfied. For relatively risk- free firms, it should
be nearly the same as the interest rate on risk-free government bonds.
 Short run: period of time for which two conditions hold: The firm is
operating under a fixed scale (fixed factor) of production, and firms can
neither enter nor exit an industry.
 Long run: period of time for which there are no fixed factors of production:
Firms can increase or decrease the scale of operation, and new firms can
enter and existing firms can exit the industry.
 The Bases of Decisions: (1) The market price of output which determines
potential revenues, (2) The techniques of production that are available, and
(3) The prices of inputs.
 Production function or total product function is a numerical or

Team of Microeconomics I Teaching Assistant | 33


mathematical expression of a relationship between inputs and outputs.
 Marginal product is the additional output that can be produced by adding
one more unit of a specific input, ceteris paribus.
∆𝑇𝑃 𝑇𝑃2−𝑇𝑃1
 MP = = , where X is input
∆𝑋 𝑋2−𝑋1

 Law of diminishing returns occurs when additional units of a variable input


are added to fixed inputs after a certain point, the marginal product of the
variable input declines.
 Average product is the average amount produced by each unit of a variable
factor of production.
 Two things determine the cost of production:
- Technologies that are available and
- Input prices.

Team of Microeconomics I Teaching Assistant | 34


CHAPTER 4
THE PRODUCTION PROCESS

Fill In The Blank

1. Profit (economic profit) is the difference between ________ and________.

2. Changing the ________ of production will change the relationship between


input and output quantities

3. Total revenue of TAMIMA Firm is $5400 and its total cost is $3700. So,
the profit that will TAMIMA Firm get is ________.

4. Short run decision is a condition when a firm is operating under a


________scale of production, and firms can neither enter nor exit an
industry.

5. A graph that shows the combination of two inputs for a given level of
production output is called ________.

6. ________ is the annual flow of net income generated by an investment


expressed as a percentage of the total investment

7. Marginal product is an additional ________that can be produced by adding


one more unit of a specific input, ceteris paribus.

8. Production is the process through which inputs are combined and


transformed into ________.

9. The relationship between inputs and outputs expressed numerically or


mathematically is called a ________.

Team of Microeconomics I Teaching Assistant | 35


10. The law of diminishing returns states that after a certain point, when
additional units of a variable input are added to fixed inputs, the marginal
product of the variable input ________.

True or False

1. Profit-maximizing firms will choose the technology that minimizes the cost of
production given current market input prices.

2. Firms in an economy with high labor costs have an incentive to use Labor-
intensive technologies.

3. Marginal product and average product are equal when marginal product is at
its maximum.
4. Total product reaches its highest level where marginal product is equal to
average product.
5. Capital-intensive technology is technology that relies heavily on human labour
instead of capital.
6. When marginal product is equal to zero, average product is rising.
7. In the short run, the firm operates under fixed cost.

8. The relationship between the average product of labor (AP L) and the marginal
product of labor (MPL) is When MPL is above APL, APL rises.
9. When marginal product is above average product, average product is rising.
10. Profit-maximizing firms will minimize costs by producing their chosen level of
output with the technology represented by the point at which the isoquant is
tangent to an isocost line.

Team of Microeconomics I Teaching Assistant | 36


Multiple Choice

1. These are three decisions that all firms make to achieve maximum profit,
except ………

a. How much output to supply.


b. Which production technology to use.
c. How much of input to demand.
d. How much output demanded.

2. We can define the Average Product of Capital mathematically:

a. AP = TP
L

b. AP = TP
K

c. AP = ∆TP
∆L

d. AP = ∆TP
∆K

3. These conditions are in the long run decision, except .........


a. There are no fixed factors of production
b. Firms can enter the industry
c. Firms cannot exit the industry
d. Firms can increase the scale of operation

4. Firms with high capital costs in an economy have an incentive to use .........

a. Labour-intensive technologies.

Team of Microeconomics I Teaching Assistant | 37


b. Capital-intensive technologies.
c. Less than optimal production technologies.
d. The production method than maximises cost.

5. In competitive market each buyer and seller .........


a. Is price maker
b. Is price taker
c. Do product differentiation
d. Has market power

6. Based on this figure, which point that cost minimizing equilibrium


condition for the firm?

a. Point C.
b. Point D.
c. Point D, C, and B.
d. Point D and B.

7. Marginal Product will equal to Average Product when .........

Team of Microeconomics I Teaching Assistant | 38


a. Average product minimum.
b. Average product maximum.
c. Marginal product maximum.
d. Marginal product minimum.

8. Suppose that L1 = 0, L2 = 1, L3 = 2, and L4 = 3. Then, we have TP1 = 0, TP2


= 10, TP3 = 25, and TP4 = 35. What will happen to its marginal product?

a. It will be increasing.
b. It will be decreasing.
c. It will be the same at 4 point.
d. None the above answer is true.

9. We can get profit from the difference between ......... and .........
a. Total revenue, total cost
b. The sale of the product, total input
c. Pocket cost, opportunity cost
d. Marginal revenue, marginal cost

10. Which of the following statements is correct?

Team of Microeconomics I Teaching Assistant | 39


a. At points D, C, and B, the total cost of production is minimized.
b. Points D, C, and B show different combinations of inputs that yield
the same cost of production.

c. At points D, C, and B, the amount of output produced is the same.

d. All of the above.

Essay

1. Explain the difference condition that will be taken by a firm in the short run
and long run!
2. Consider a firm that uses capital and labor as inputs and sells 5,000 units of
output per year at the going market price of $10. Also assume that total labor
costs to the firm are $45,000 annually. Assume further that the total capital
stock of the firm is currently worth $100,000, that the return available to
investors with comparable risks is 10 percent annually, and that there is no
depreciation. Is this a profitable firm? Explain your answer

Team of Microeconomics I Teaching Assistant | 40


3. The following table gives total output or total product as a function of labor
units used.

a. Define diminishing returns.


b. Does the table indicate a situation of diminishing returns?
Explain your answer.
4. Suppose that widgets can be produced using two different production
techniques, A and B. The following table provides the total input
requirements for each of five different total output levels.

a. Assuming that the price of labor (PL) is $1 and the price of capital
(PK) is $2, calculate the total cost of production for each of the
five levels of output using the optimal (leastcost) technology at
each level.
b. How many labor hours (units of labor) would be employed at each
level of output? How many machine hours (units of capital)?
c. Graph total cost of production as a function of output. (Put cost on
the Y-axis and output, q, on the X-axis.) Again assume that the
optimal technology is used.

Team of Microeconomics I Teaching Assistant | 41


d. Repeat a. through c. under the assumption that the price of labor
(PL) rises from $1 to $3 while the price of capital (PK) remains at
$2.
5. A firm earning zero economic profits is probably suffering losses from the
standpoint of general accounting principles. Do you agree or disagree with
this argument? Explain why.
6. Define Isoquant and Isocost using graph and explanation!
7. The following is a production function.

a. Draw a graph of marginal product as a function of output. (Hint:


Marginal product is the additional number of units of output per
unit of labor at each level of output.)
b. Does this graph exhibit diminishing returns? Explain your answer
8. A firm can use three different production technologies, with capital and
labor requirements at each level of output as follows :

Team of Microeconomics I Teaching Assistant | 42


a. Suppose the firm is operating in a high-wage country, where
capital cost is $100 per unit per day and labor cost is $80 per
worker per day. For each level of output, which technology is
cheapest?
b. Now suppose the firm is operating in a low-wage country, where
capital cost is $100 per unit per day but labor cost is only $40 per
unit per day. For each level of output, which technology is
cheapest?
c. Suppose the firm moves from a high-wage to a low-wage country
but its level of output remains constant at 200 units per day. How
will its total employment change?
9. The following table represents data for Samantha’s Smoothies. Draw a
graph showing the total product, marginal product of labor, and average
product of labor. Identify where increasing returns, diminishing returns and
negative returns set in on the total product curve.

Team of Microeconomics I Teaching Assistant | 43


10. Which of the following are short-run decisions and which are long-run
decisions?
a. General Motors decides to add a second shift to its Arlington,
Texas production plant.
b. Gotham Foods International chooses to exit the restaurant industry
to concentrate on its wholesale grocery supply business.
c. The Sahara Hotel and Casino in Las Vegas closes two of its three
hotel towers in response to low demand.
d. Tony Andretti, owner of Tony the Taxman, hires five new CPAs
to work at his tax preparation business.
e. German tool and appliance manufacturer Bosch enters the electric
bicycle industry in 2010.

Team of Microeconomics I Teaching Assistant | 44


CHAPTER 5
SHORT RUN COSTS AND OUTPUT DECISIONS

• There are three decisions facing firms:


a. The quantity of output to supply
b. How to produce that output (which technique to use)
c. The quantity of each input to demand
• The short run is a period of time for which two conditions hold:
1. The firm is operating under a fixed scale (fixed factor) of production,

2. Firms can neither enter nor exit an industry.


• Costs in the short run :
Fixed cost is any cost that does not depend on the firm’s level of output.
These costs are incurred even if the firm is producing nothing.

Variable cost is a cost that depends on the level of production chosen.


TC = TFC + TVC
Where, TC : Total Costs
TFC : Total Fixed Cost
TVC : Total Variable Cost
Average Fixed Cost (AFC), is the total fixed cost (TFC) divided by the
number of units of output (q).

Average Variable Cost (AVC), is the total variable cost (TVC) divided by the
number of units of output (q).

Marginal Cost (MC), is the increase in total cost that result from producing
one more unit of output. Marginal cost reflects changes in variable costs.

Team of Microeconomics I Teaching Assistant | 45


• Causes marginal cost is the cost of one additional unit. Average variable
cost is the average variable cost per unit of all units being produced. So,
average variable cost follows marginal cost, but lags behind.

• Sunk costs occur when firms have no control over fixed costs in the short
run.

• The fact, in the short run the firm faces diminishing returns to variable
inputs and the firm has limited capacity to produce output.

• The relationship between marginal cost and average variable cost :


a. When marginal cost is below average cost, AC is declining.
b. When marginal cost is above average cost, AC is increasing.
• Total Revenue is the total amount that a firm takes in from the sale of its
output.

TR = P x q
• Marginal Revenue is the additional revenue that a firm takes in when it
increases output by one additional unit.

• In perfect competition, MR = P, therefore, the profit-maximising perfectly


competitive firm will produce up to the point where the price of its output
is just equal to short run marginal cost. The key idea is that firms will
produce as long as marginal revenue exceeds marginal cost.

Team of Microeconomics I Teaching Assistant | 46


CHAPTER 5
SHORT RUN COSTS AND OUTPUT DECISIONS

Fill In the Blank

1. Total cost is ________ cost plus ________ cost.


2. Total variable cost divided by the number of units of output is called _____.

3. Additional cost caused by the additional of one unit being produced


called_____.

4. The demand curve in _____________ market is horizontal.


5. Average total cost is declining when marginal cost is _____ the average
total cost.

6. In perfect competitive market, price is equal to _____.


7. A firm will keep producing output as long as marginal revenue exceeds
_____.

8. Suppose that a firm produces 400 units of book with cost incurred about
$3000, and the price of each book is $8, the profit that the firm get will be
_____.

9. When total cost is exceeds the total revenue then the firm is suffering
_____.

10. A marginal cost is the first derivation of _____________.

True or False

1. In the long run a firm can not enter and exit freely.

Team of Microeconomics I Teaching Assistant | 47


2. The maximum profit in perfect competitive market occurs when Marginal
Revenue exceeds the Marginal Cost.
3. Marginal cost intersects the average total cost and average variable cost
curves at the same points.

4. Sunk costs occur when firms have control over fixed costs in the short run.

5. When ATC declines, AVC will increasing .


6. When marginal cost is above average cost, it means that the average cost is
declining.

7. Firms want to maximize the difference of their total revenue and total cost.

8. ATC curve is always increasing together with output.


9. Total marginal cost always has negative slope curve.
10. The average total cost curve of a perfectly competitive profitmaximizing
firm is the firm’s short-run supply curve.

Multiple Choice

1. If PT. ECON produced 1,000 pencils with $55,000 total variable cost, how
much is the average variable cost?

a. $550
b. $55
c. $54,000
d. zero

2. The relationship between average cost and marginal cost is as follows:

a. Marginal cost is below average cost when average cost rises


b. Marginal cost is below average cost when average cost declines

Team of Microeconomics I Teaching Assistant | 48


c. Marginal cost is above average cost when average cost declines
d. Marginal cost is above average cost when average cost is at a
minimum

3. The profit-maximizing level of output in perfect competition for all firms is


where ...

a. P = MC
b. AC > MC
c. MR < MC
d. MR = AVC

4. Refer to the figure below, suppose that the market is initially in equilibrium
at a price of $5. Then, market demand increases to D1.

What should the firm do?

a. Maintain the level of output constant at 300 units, where ATC is


minimum, this will guarantee maximum profit.

b. Produce 300 units but charge the higher price of 6 in order to


maximize profit.

Team of Microeconomics I Teaching Assistant | 49


c. Increase the level of output to 350, although it is unlikely that profit
will increase.
d. Increase the level of output to 350 units because profit would increase.

5. Suppose that Dauntless Inc. produced 850 unit of output and get profit
$1,850. The total cost of that firm is $5800. How much the price of each
output are being sold?

a. $7
b. $9
c. $11
d. $13

6. Refer to the figure below. A phenomenon often called spreading overhead


is better illustrated by one of the graphs below. Which one?

a. Both graphs illustrate that phenomenon.


b. Neither graph illustrates that phenomenon.
c. The graph on the right.
d. The graph on the left.

Team of Microeconomics I Teaching Assistant | 50


7. Which of the following curves embodies information about both input
prices and technology?

a. The average fixed cost curve.


b. The total variable cost curve.
c. The total fixed cost curve.
d. All of the above.

8. Marginal cost intersects the and curves at their minimum


points.

a. Total variable cost; Average fixed cost.


b. Average variable cost; Average fixed cost.
c. Total fixed cost; Average variable cost.
d. Average total cost; Average variable cost.

9. Firms have no control of fixed costs in the short run; they have no choice
but to pay them. For this reason, fixed costs are sometimes called ............

a. Average fixed costs.


b. Sunk costs.
c. Short run costs.
d. Long run costs.

10. These are the characteristics of perfect competitive market, except ..........

a. prefect information
b. has large number of sellers and buyers
c. price maker
d. free entry and exit industry

Team of Microeconomics I Teaching Assistant | 51


Essay

1. Do you agree or disagree with this statement? Firms minimize costs;


thus, a firm earning short-run economic profits will choose to produce
at the minimum point on its average total cost function.

2. Explain the concept below:


a. Sunk cost
b. Spreading overhead
c. Marginal cost
3. Determine the best technique used by the company at every level of output
in order to maximize profits earned? (if P k=2; PL=3)

Product Technique Units of Input TVC


K L
1 unit output A 6 4

B 4 6
2 unit output A 10 8
B 8 10
3 unit output A 11 6

B 7 12

4. What is the relationship between marginal cost curve with average total cost
and average variable cost? Explain and draw the diagram.

5. Why the marginal cost curve U-shaped? Explain clearly (plus you must
show it in table and graph).
6. Complete the table below! (and if firm set price at P*)

Team of Microeconomics I Teaching Assistant | 52


Q TC TFC TVC AVC ATC MC P* TR ∏
0 110 40 60 ~ ~ - 50 0 (110)
1 140 40 90 90 140 30 50 60 (80)
2 160 40 110 55 80 20 50 120 (40)
3 170 40 120 40 56.67 10 50 180 10
4 182 40 132 33 45.5 12 50 240 58
5 195 40 145 29 39 13 50 300 105
6 220 40 170 36.67 36.67 25 50 360 140
7 250 40 200 28.57 35.71 30 50 420 170
8 290 40 240 30 36.25 40 50 480 190
9 340 40 290 32.22 37.77 50 50 540 200
10 400 40 350 35 40 60 50 600 200

7. For each of the following businesses, what is the likely fixed factor of
production that defines the short run?

a. Potato farm of 160 acres


b. Chinese restaurant
c. Dentist in private practice
d. Car dealership
e. Bank
8. Based on the information on the graph, fill in the blanks in the table below!

Team of Microeconomics I Teaching Assistant | 53


Output TVC TFC TC
100
300
400

9. Which the following statements involves a short-run decision and which


involves a long-run decision? Explain briefly!

a. McD will open 89 more stores abroad than originally predicted, for a
total of 1340.

b. For three hours on Tuesday, Starbucks will shut down every single
one of its 7,100 stores so that baristas can receive a refresher course.

c. In the Ramadhan month, the PT. Erudite impose an office hours only
from 9.00 am until 4.00 pm.

d. Wendy’s is closing 190 stores by the end of April.

Team of Microeconomics I Teaching Assistant | 54


10. You are given the following cost data:
Total fixed costs are 10.
q TVC
0 0
1 5
2 15
3 25
4 40
5 65
6 95
If the price of output is $15, how many units of output will this firm
produce? Then, what is total revenue? What is total cost? Briefly
explain using the concept of marginal cost. What do you think the firm
is likely to do in the short run?

Team of Microeconomics I Teaching Assistant | 55


CHAPTER 6
LONG RUN COSTS & OUTPUT DECISIONS

• We begin our discussion of the long run by looking at firms in three short-
run circumstances:

(1) firms earning economic profits,


(2) firms suffering economic losses but continuing to operate to reduce or
minimize those losses, and

(3) firms that decide to shut down and bear losses just equal to fixed costs.

• In general,
(1) If revenues exceed variable costs, operating profit is positive and can
be used to offset fixed costs and reduce losses, and it will pay the firm
to keep operating.

(2) If revenues are smaller than variable costs, the firm suffers operating
losses that push total losses above fixed costs. In this case, the firm
can minimize its losses by shutting down.

• Shut-down point is the lowest point on the average variable cost curve.
When price falls below the minimum point on AVC, total revenue is
insufficient to cover variable costs and the firm will shut down and bear
losses equal to fixed costs.

• Short-run industry supply curve is the sum of the marginal cost curves
(above AVC) of all the firms in an industry.

Team of Microeconomics I Teaching Assistant | 56


• Increasing returns to scale, or economies of scale. An increase in a firm’s
scale of production leads to lower costs per unit produced.

• Constant returns to scale. An increase in a firm’s scale of production has


no effect on costs per unit produced.

• Decreasing return to scale, or diseconomies of scale. An increase in a


firm’s scale of production leads to higher costs per unit produced.

Team of Microeconomics I Teaching Assistant | 57


CHAPTER 6
LONG RUN COSTS & OUTPUT DECISIONS

Fill in the Blank

1. In the long run , the firm can increase any or all of its inputs and thus have no
_____ ____ of ______ that confines its production to a given scale.
2. Firms are free to _____ industries to seek ____ and to ___ industries to _____
losses.
3. If exceeds total variable cost, the excess can be used to pay some fixed
costs and thus reduce losses.
4. Long-run competitive equilibrium is reached when P = = =
and profits are zero.
5. The short-run supply curve of a firm in a perfectly competitive industry is the
portion of its curve that lies above its curve.
6. ___________ is the smallest output at which long run average cost reaches its
lowest level.
7. In the short run, the quantity of at least one factor of production is ____ and the
quantities of the other factors of production can be _____.
8. With economies of scale, the long run average cost curve slopes _______.
9. As long as economic profits are being earned in an industry, firms will _____
the industry and the supply curve will shitf to the _____
10. In the long run firms will expand as long as there are more ________

Team of Microeconomics I Teaching Assistant | 58


True or False

1. profit includes total revenue minus total cost, AND total cost include a normal
rate of return
2. Shutdown Point is the situation in which each firm is earning exactly a normal
rate of return
3. Constant Returns to Scale is a condition where an increase in a firm's scale of
production has no effect on costs per unit produced
4. Economic cost curves define the minimum economic costs of producing various
levels of output.
5. All costs are variable costs in the long run.
6. Firms that produce more than one type of product cannot benefit from
economies of scope.
7. If marginal cost is greater than average total cost, then average total cost is
rising.
8. If the long-run average cost curve slopes upward over some range of output,
then the firm is experiencing increasing returns to scale over that range of
output.
9. The law of diminishing returns is reflected in the downward-sloping portion of
the short-run marginal cost curve.
10. The entrepreneur's opportunity cost is an implicit cost.

Team of Microeconomics I Teaching Assistant | 59


Multiple Choice

1. Which of the following is the shut-down point?


a. The lowest point on the average total cost curve
b. The point where price equals average total cost at the profit
maximizing level of output
c. The lowest point on the average fixed cost curve.
d. The lowest point on the average variable cost curve.
2. Which of the following values cannot be calculated at the firm's breakeven
level of output?
a. operating leverage.
b. contribution margin per unit.
c. degree of operating leverage.
d. profit.
3. The short-run industry supply curve is the sum of the individual firm supply
curves-that is ....

a. The marginal cost curves (above AVC) of all the firms in the
industry
b. The average total curves (above price) of all the firms in the
industry
c. The horizontal demand curves (at the price level) of all the firms
in the industry
d. The average variable cost curves (above price) of all firms in the
industry
4. Refer to the figure below. Which of the firms below chooses to produce output
at a loss?

Team of Microeconomics I Teaching Assistant | 60


a. A, C, and D

b. C

c. A

d. Both A andC

5. if a firm has a downward sloping long-run average cost curve, then


a. it is experiencing decreasing returns to scale.
b. it is experiencing decreasing returns.

Team of Microeconomics I Teaching Assistant | 61


c. it is a natural monopoly.
d. marginal cost is greater than average cost.
6. The contribution margin per unit is equal to the
a. price of a good.
b. the difference between total revenue and total cost.
c. difference between price and average total cost.
d. difference between price and average variable cost.

7. When a firm breaks even, which of the following occurs?

a. Both profit and the rate of return equal zero


b. Profit is positive but the firm may earn a rate of return below normal
c. Profit is zero. The firm earns exactly the normal rate of return
d. Profit is zero but the rate of return could be above or below the normal
rate of return
8. Whether or not a firm decides to produce or shut down in the short run depend
solely on whether revenues from operating are sufficient to cover ......

a. Total costs
b. Fixed costs
c. Variable costs
d. Normal profit

9. The law of diminishing returns begins at the level of output where


a. marginal cost is at a minimum.
b. average variable cost is at a minimum.
c. average fixed cost is at a maximum.
d. None of the above is correct.

Team of Microeconomics I Teaching Assistant | 62


10. Refer to the figure below. When market price equals $20, the profit
maximizing

a. Produces 500 units of output and earns economic profit of $5000


b. Produces 500 units of output and earns economic profit of $4000
c. Produces 320 units of output and earns economic profit of $3200
d. Produces 320 units of output and earns economic profit of $10

Essay

1. Explain the relationship between marginal product and average product!


2. How the circumstances of the firm in the short-run ?
3. Draw short run curve of total production (TP), average product (AP), and

Team of Microeconomics I Teaching Assistant | 63


marginal product (MP). Explain the definition of each curve!
4. This table below shows the total production of company A.
Labour 1 2 3 4 5
Output 6 14 24 32 28

If fixed cost (FC) is Rp 1000,- and wage per labor is Rp 500,-, make a table
of TFC, TVC, TC, ATC, AVC, AFC, and MC. The draw each curve!
5. Explain about economies and diseconomies of scale!
6. The long-run average cost curve for an industry is represented in the
following graph. Add short-run average cost curves and short-run
marginal cost curves for three firms in this industry, with one firm
producing an output of 10,000 units, one firm producing an output of
20,000, and one firm producing an output of 30,000. Label these as Scale
1, Scale 2, and Scale 3, respectively. What is likely to happen to the scale
of each of these three firms in the long run?
7. On the following graph for a purely competitive industry, Scale 1
represents the short-run production for a representative firm. Explain what
is currently happening with firms in this industry in the short run and what
will likely happen in the long run.

Team of Microeconomics I Teaching Assistant | 64


8. For each of the following, decide whether you agree or disagree and
explain your answer:
a. A firm earning positive profits in the short run always has an
incentive to increase its scale of operation in the long run.
b. A firm suffering losses in the short run will continue to operate
as long as total revenue at least covers fixed cost.
9. Explain why it is possible that a firm with a production function that
exhibits increasing returns to scale can run into diminishing returns at the
same time.
10. The shape of a firm’s long-run average cost curve depends on how costs
vary with scale of operation. Draw a long-run average cost curve for a
firm that exhibits economies of scale, constant returns to scale, and
diseconomies of scale. Identify each of these sections of the cost curve
and explain why each section exemplifies its specific type of returns to
scale.

Team of Microeconomics I Teaching Assistant | 65


CHAPTER 7
INPUT MARKET

• Input market is households supply inputs to all resource market; firms


demand in all resource market. Three important inputs are labor, land
and capital. Labor gets wage, land gets rent and capital gets interest

• Inputs can be complementary or substitutable. Two inputs used together


may enhance, or complement each other.

• Marginal product of labor (MPL) is the additional output produced by 1


additional unit of labor.

• Marginal revenue product (MRP) is the additional revenue a firm earns


by employing 1 additional unit of input, ceteris paribus. MRPL

= MPL . PX
• Factor substitution effect is the tendency of firms to substitute away
from a factor whose price has risen and toward a factor whose price has
fallen. Demand-determined price is the price of a good that is in fixed
supply; it is determined exclusively by what firms and households are
willing to pay for the good.

• Produced on that land is sufficient to cover the price of the land. Stated
in equation form, the firm will use land up to the point at which MRPA=
PA, where A is land

• Profit-maximizing condition for the perfectly competitive firm is :


PL = MRPL = (MPL x PX)
PK = MRPK = (MPK x PX)
PA = MRPA = (MPA x PX)

Team of Microeconomics I Teaching Assistant | 66


• Physical or tangible capital is material things used as inputs in the
production of future goods and services. The major categories of
physical capital are nonresidential structures, durable equipment,
residential structures, and inventories.

• Intangible capital is nonmaterial things that contribute to the output of


future goods and services. human capital A form of intangible capital
that includes the skills and other knowledge that workers have or
acquire through education and training and that yields valuable services
to a firm over time.

• Investment is new capital additions to a firm’s capital stock. Although


capital is measured at a given point in time (a stock), investment is
measured over a period of time (a flow). The flow of investment
increases the capital stock. Depreciation is the decline in an asset’s
economic value over time.

• The expected benefit of investments is the investment process requires


that the potential investor evaluate the expected flow of future
productive services that an investment project will yield.

• The expected cost of investments is the ability to lend at the market rate
of interest means that there is an opportunity cost associated with every
investment project. The evaluation process thus involves not only
estimating future benefits but also comparing them with the possible
alternative uses of the funds required to undertake the project.

• Expected rate of return is the annual rate of return that a firm expects to
obtain through a capital investment. The expected rate of return on an
investment project depends on the price of the investment, the expected
length of time the project provides additional cost savings or revenue,
and the expected amount of revenue attributable each year to the project.

Team of Microeconomics I Teaching Assistant | 67


• A perfectly competitive profit-maximizing firm will keep investing in
new capital up to the point at which the expected rate of return is equal
to the interest rate. This is analogous to saying that the firm will
continue investing up to the point at which the marginal revenue product
of capital is equal to the price of capital, or MRPK = PK,

• Present value (PV) is the present discounting value of R dollars to be


paid t years in the future is the amount you need to pay today, at current
interest rates, to ensure that you end up with R dollars t years from now.
It is the current market value of receiving R dollars in t years.

Team of Microeconomics I Teaching Assistant | 68


CHAPTER 7
INPUT MARKET
Fill in the Blank

1. Marginal Product is the amount of ________ produced per unit of


__________
2. When the Output price is higher, the labor demand curve will ________ to
the ________
3. Some factors that might shift the labor supply curve are _________ ,
___________, and _________.
4. When the Marginal Revenue Product of Labor is lower than the Price, the
Firm will _________ their demand of labor.
5. As individuals get richer, the marginal utility of a dollar will _________,
which shift the labor supply to ___________
6. When the MRPL is 100 and Marginal Productivity of Labor is 25 unit,
suppose the firm act rationally, it will set price in _________
7. When the land and labor are used together complementary, with fewer
workers available, an additional unit of land produced _______ additional
output, or marginal productivity of land _____________
8. When price of capital is lower than marginal revenue product of capital,
The firm will ________ their invest until the point that optimize their
profit, which is __________.
9. The expected rate of return on an investment depend on price of
investment, _____________________, and _____________________
each year of the project. Expected length of time (maturity).

10. The decline in an asset’s economic value over time is called


_____________. So, you have to execute your investment projects as soon
as possible.

Team of Microeconomics I Teaching Assistant | 69


True or False

1. The concept of derived demand states that demand of output those resources
can be used to produce is depended on the demand for inputs by firm.
2. In n output, When the Marginal Product of Labor is 1000, Price is $20, and
Wage Cost is $21000, the Marginal Profit in n output is negative
3. Technological advance will raises the marginal product of labor, which its
positive effect will only increases the demand of labor and shifts it to the
4. An event that changes the supply of any factor of production can alter the
earnings of all factors
5. The return of the land is supply-determined, depend on what’s the supply of
land available in the market.
6. When the reduced population happened, with a smaller supply of workers, the
marginal product of labor will decreasing.
7. When the maturity or length time of investment is longer, the rate of return
will be higher.
8. The supply of land is very fixed. Otherwise, the demand of land has is
perfectly elastic
9. When the interest rate of bond higher, the supply of capital will be higher
10. The sophisticated engine with electricity power are complementary inputs. The
increase in electricity tariff will reduce demand for labor.

Team of Microeconomics I Teaching Assistant | 70


Multiple Choice

1. Refer to the figure, what explains the shape of this curve in framework of
competitive input and output markets?

a. Increasin output price, decreasing marginal product of labor and


diminishing return
b. Decreasing marginal product of labor along with lower output price
c. Increasing output price, decreasing marginal product of labor, and
diminishing return
d. Diminishing marginal returns, consequently, decreasing marginal
product of labor, but not output price.

2. A technological advance that increases the marginal


product of labor shifts the labor- __________ curve to the
a. demand, left
b. demand, right
c. supply, left
d. supply, right

Team of Microeconomics I Teaching Assistant | 71


3. If firms are competitive and profit-maximizing, the demand curve for
labor is determined by
a. the opportunity cost of workers’ time.
b. the value of the marginal product of labor.
c. offsetting income and substitution effects.
d. the value of the marginal product of capital.
4. A storm destroys several factories, thereby reducing the stock of capital.
What effect does this event have on factor markets?
a. Wages and the rental price of capital both rise.
b. Wages and the rental price of capital both fall.
c. Wages rise, and the rental price of capital falls.
d. Wages fall, and the rental price of capital rises.

5. A bakery operating in competitive markets sells its output for $20 per
cake and hires labor at $10 per hour. To maximize profit, it should hire
labor until the marginal product of labor is (P = MRPL (MPL.P) -> 20 =
MPL.10
a. 1/2 cake per hour.
b. 2 cakes per hour.
c. 10 cakes per hour.
d. 15 cakes per hour.
6. In order to optimize profit, firm will hire workers up until
a. P = MC
b. MR = MC
c. W = MRPL
d. W = MPL

Team of Microeconomics I Teaching Assistant | 72


7. The function of interest and profut are
a. Benefits of delaying consumption and benefit of innovation and
decision making
b. Benefits of delaying consumption and benefits of optimizing profit
c. Benefits of income and benefits of decision making
d. Benefits of storing wealth and benefits of optimizing profit
8. The demand for new capital depends on interest rate. When the intrest rate
is …………., firms are less likely to invest in new plant and equipment
than when there interest rate is ………
a. Low, Low
b. Low, High
c. High, High
d. High, Low
9. When the maturity of investment is longer, the interest rate or risk
premium will be ……… So does the money owner will …………. their
saving
a. Lower ; Decrease
b. Higher ; Increase
c. Lower ; Increase
d. Higher ; Decrease
10. The input prices of labor, kapital and land are $8, $16, and $22. The
Marginal Product of each input are 12. 24, and 33 units. What s the price
of good Y to achieve optimum condition of the input?
a. 0,33 $
b. 2$
c. 1,5 $
d. 1,33 $

Team of Microeconomics I Teaching Assistant | 73


Essay

1. A medium-sized bakery has just opened in Slovakia. A loaf of bread is


currently priced for 14 Koruna (Slovakian Currency), above the cost of its
ingredients. Assumed that labor is the only variable factor of production,
the following table gives the production function of bread

WORKERS 0 1 2 3 4 5 6 7
BREAD 0 15 30 42 52 60 66 70
a. Suppose that current wage rate in Slovakia is 119 Koruna/hour. How
many workers will the bakery employ
b. Suppose that the economy of Slovakia begins to grow. Income rises,
and the price of a bread is 20 koruna now. With no increase in the
wage, how many workers will bakery hire?
c. An increase in labor demand pushes up wages to 125 koruna/hour.
What impact will this increase in cost of have an employment and
output in the bakery at the 20-koruna price of bread?

2. Look carefully the flow of income in each the end of year below

End Agung Holcim NET Yamaha Paytren


Year Podomoro
1 $ 120 40 150 100 750
2 120 40 150 100 450
3 120 40 1650 100 600
4 120 40 0 100 450
5 1620 40 0 100 0
6 0 40 0 110 0
7 0 540 0 0 0

a. Calculate the present value of income flow with interest rates


: 8 % and 10 %

b. Suppose that the investment cost of Paytren is $ 1853 in


initial year.Will you put your investment when the interest
rate is 8 %? How about 10 % ?

Team of Microeconomics I Teaching Assistant | 74


3. There is a firm that using two inputs. The quantities of production are
below :

Land 1 1 1 1 1 1 1 1 1 1
Labor 0 1 2 3 4 5 6 7 8 9
Total 0 2 5 9 12 14 15 15 14 12
Production
a. Please find the average product and marginal product from labor
b. Graph the curves of total product, average product, and marginal
product
c. Please identify three stages of scale production from these quantity of
labor
d. In the graph, where are the point that show these production elasticity
(EP) ; EP>1 ; EP<1 ; EP = 1 and EP=0
4. Explain the differences between briefly :
a. Profit and Interest Rate
b. Marginal Product of Input and Marginal Revenue Product of Input
c. Factor substitution effect and output effect
5. Technological advance have two effects for demand of input by the firm.
Please explain those effects briefly.
6. Which of the following are capital and not, explain your answer briefly
a. Individual saving account at Panama Bank.
b. Ibnu buys Yamaha Motorcycle YZR-M1 in ebay for his collection.
c. Stadion Utama Gelora Bung Karno as the home base of Persija
Jakarta for Football Match
d. The Collection of Museum Geologi in Bandung.
e. Ambulance Medic Training and Certification from the Red Cross.

7. Give at least three examples of how savings can be channeled into


productive investment. Why is investment so important for an economy?
What do you sacrifice when you save today?

8. Show the effect of each of the following events on the market for labor in
the computer manufacturing industry.
a. Congress buys personal computers for all U.S. college students.

Team of Microeconomics I Teaching Assistant | 75


b. More college students major in engineering and computer science.
c. Computer firms build new manufacturing plants
9. Suppose that labor is the only input used by a perfectly competitive firm.
The firm’s production function is as follows:
Days of Units of Output
Labor
0 0
1 7
2 13
3 19
4 25
5 28
6 29
7 29

a. Calculate the marginal product for each additional worker.


b. Each unit of output sells for $10. Calculate the value of the marginal
product of each worker.
c. Compute the demand schedule showing the number of workers hired
for all wages from zero to $100 a day.
d. Graph the firm’s demand curve.
e. What happens to this demand curve if the price of output rises from
$10 to $12 per unit?

10. Policymakers sometimes propose laws requiring firms to give workers


certain fringe benefits, such as health insurance or paid parental leave.
Let’s consider the effects of such a policy on the labor market
a. Suppose that a law required firms to give each worker $3 of fringe
benefits for every hour that the worker is employed by the firm. How
does this law affect the marginal profit that a firm earns from each
worker at a given cash wage? How does the law affect the demand
curve for labor? Draw your answer on a graph with the cash wage on
the vertical axis.
b. If there is no change in labor supply, how would this law affect
employment and wages?

Team of Microeconomics I Teaching Assistant | 76


c. Why might the labor-supply curve shift in response to this law?
Would this shift in labor supply raise or lower the impact of the law
on wages and employment?
d. The wages of some workers, particularly the unskilled and
inexperienced, are kept above the equilibrium level by minimum-
wage laws. What effect would a fringe-benefit mandate have for these
workers?

Team of Microeconomics I Teaching Assistant | 77


CHAPTER 8
MONOPOLY MARKET, OLIGOPOLY MARKET, GAME
THEORY & MONOPOLISTIC MARKET

• Imperfectly competitive industry is an industry in which individual


firms have some control over the price of their output.

• Market power is an imperfectly competitive firm’s ability to raise


price without losing all of the quantity demanded for its product.

• A monopoly is a firm that is the sole seller in its market. A monopoly


arises when a single firm owns a key resource, when the government
gives a firm the exclusive right to produce a good, or when a single firm
can supply the entire market at a smaller cost than many firms could.

• When a monopoly increases production by 1 unit, it causes the price of


its good to fall, which reduces the amount of revenue earned on all units
produced. As a result, a monopoly’s marginal revenue is always below
the price of its good. And the profit maximum is when MR=MC.

• Monopolist’s revenue :
- Total Revenue
P Q = TR
- Average Revenue
TR/Q = AR = P
- Marginal Revenue
DTR/DQ = MR

• Price discrimination is the business practice of selling the same good at


different prices to different customers, even though the costs for
producing for the two customers are the same.

Team of Microeconomics I Teaching Assistant | 78


• Two important effects of price discrimination:
- It can increase the monopolist’s profits. - It can reduce
deadweight loss.

• A monopolistically competitive market is characterized by three


attributes: many firms, differentiated products, and free entry. Because
monopolistically competitive firms produce differentiated products,
each firm advertises to attract customers to its own brand.

To some extent, advertising manipulates consumers’ tastes, promotes


irrational brand loyalty, and impedes competition. To a larger extent,
advertising provides information, establishes brand names of reliable
quality, and fosters competition.

• Products can be differentiated horizontally or vertically. Horizontal


differentiation produces different types of a good with different appeals
to different types of people. In vertical differentiation, people agree that
one product is better than another; they just may not be willing to pay
for the better good.

• An oligopoly is an industry dominated by a few firms that, by virtue of


their individual sizes, are large enough to influence market price. The
behaviour of a single oligopolistic firm depends on the reactions it
expects of all the other firms in the industry. Industrial strategies usually
are very complicated and difficult to generalize about.

• When firms collude, either explicitly or tacitly, they jointly maximize


profits by charging an agreed-to price or by setting output limits and
splitting profits.

• The price-leadership model of oligopoly leads to a result similar but


not identical to the collusion model. In this organization, the dominant

Team of Microeconomics I Teaching Assistant | 79


firm in the industry sets a price and allows competing firms to supply
all they want at that price. An oligopoly with a dominant price leader
will produce a level of output between what would prevail under
competition and what a monopolist would choose in the same industry.

• The Cournot model of oligopoly is based on three assumptions: (1) that


there are few firms in an industry, (2) that each firm takes the output of
the other as a given, and (3) that firms maximize profits.

• Game theory analyse the behavior of firms as if their behavior were a


series of strategic moves and countermoves.

• Market concentration can also lead to gains from economies of scale


and may promote innovation.

Team of Microeconomics I Teaching Assistant | 80


CHAPTER 8
MONOPOLY MARKET, OLIGOPOLY MARKET, GAME
THEORY & MONOPOLISTIC MARKET

Fill in the Blank

1. The monopolist chooses the quantity of output at which _______ equals


____ and then uses the demand curve to find the _____ that will induce
consumers to buy that quantity.
2. For the monopolist, marginal revenue is ____ than the price of the good.
3. The demand curve a monopolist is _____ curve for its product.
4. Price discrimination can raise economic welfare because _______
increases beyond that which would result under monopoly pricing.
5. Using regulations to force a natural monopoly to charge a price equal to
______ of production will cause the monopoly to lose money and exit the
industry.
6. Similar to a monopolist, a monopolistically competitive firm faces
_______ demand curve for its product. However, similar to firms in
perfectly competitive markets, firms in monopolistically competitive
markets can enter and exit the market without restriction so profits are
driven to ______ in the long run.
7. Because a monopolistically competitive firm charges a price that exceeds
marginal cost, the firm fails to. produce some units that the buyers value in
excess of the cost of production, and thus, monopolistic competition is
______
8. The unique feature of an ________ is that the actions of one seller have a
significant impact on the profits of all of the other sellers in the market.

Team of Microeconomics I Teaching Assistant | 81


9. The price and quantity by a Nash equilibrium is closer to the ______ than
the price and quantity generated by a cartel.
10. _______ demonstrates why it is difficult to maintain cooperation even
when cooperation is mutually beneficial.

True or False

1. The most common source of a barrier to entry into a monopolist's market


is that the monopolist owns a key resource necessary for production of that
good.
2. A monopolist produces an efficient quantity of output but it is still
inefficient because it charges a price that exceeds marginal cost and the
resulting profit is a social cost.
3. Perfect price discrimination is efficient but all of the surplus is received by
the consumer.
4. Monopolistic competition is a market structure in which few firms sell
similar products.
5. In the long run, firms in monopolistically competitive markets produce at
the minimum of their average total cost curves.
6. Critics of advertising argue that advertising decreases competitio.n while
defenders of advertising argue that advertising increases competition and
reduces prices to consumers.
7. Even advertising that appears to contain little information about the
product may be useful because it provides a signal about the quality of the
product.
8. An oligopoly is a market structure in which many firms sell products that
are similar but not identical.

Team of Microeconomics I Teaching Assistant | 82


9. When oligopolists collude and form a cartel, the outcome in the market is
similar to that generated by a perfectly competitive market
10. There is a constant tension in an oligopoly between cooperation and
selfinterest because after an agreement to reduce production is reached, it is
protitable for each individual firm to cheat and produce more.

Multiple Choice

1 A firm whose average total cost continually declines at least to the


quantity that could supply the entire market is known as a
a pertect competitor.
b natural monopoly.
c government monopoly.
d regulated monopoly.
2 Which of the following statements about price and marginal cost in
competitive and monopolized markets is true?
a In competitive markets, price equals marginal cost; in
monopolized equals marginal cost.
b In competitive markets, price exceeds marginal cost; in
monopolized markets, price exceeds marginal cost.
c In competitive markets, price equals marginal cost; in
monopolized markets, price exceeds marginal cost
d In competitive markets, price exceeds marginal cost; in
monopolized markets, price equals marginal cost.
3 If regulators break up a natural monopoly into many smaller firms, the
cost of production
a will fall.
b will rise.

Team of Microeconomics I Teaching Assistant | 83


c will remain the same.
d could either rise or fall depending on the elasticity of the
monopolist's supply curve.
4 If marginal revenue exceeds marginal cost, a monopolist should
a increase output.
b decrease output.
c keep output the same because profits are maximized when
marginal revenue exceeds marginal cost.
d raise the price.
5 If the monopolistic competitor described by Exhibit 3 is producing at the
profitmaximizing (loss-minimizing) level of output, it
a is generating losses.
b is generating profits.
c is generating zero profits.
d could be generating either profits or losses depending on what
quantity it chooses to produce.

Team of Microeconomics I Teaching Assistant | 84


6 The monopolistically competitive market shown in Exhibit 3 will, in the
long run,
a attract new producers into the market, which will shift the
demand faced by incumbent firms to the right.
b attract new producers into the market, which will shift the
demand faced by incumbent firms to the left.
c cause producers to exit the market, which will shift the demand
faced by incumbent firms to the right.
d cause producers to exit the market, which will shift the demand
faced by incumbent firms to the left.
7 One source of inefficiency in monopolistic competition is that
a because price is above marginal cost, surplus is redistributed
from buyers to sellers.
b because price is above marginal cost, some units are not
produced that buyers value in excess of the cost of production and this
causes a deadweight loss.
c monopolistically competitive firms produce beyond their
efficient scale.
d monopolistically competitive firms earn economic profits in the
long run.
8 When an oligopolist individually chooses its level ofproduction to
maximize its profits, it produces an output that is
a more than the level produced by a monopoly and less than the
level produced by a competitive market.
b less than the level produced by a monopoly and more than the
level produced by a competitive market.
c more than the level produced by either a monopoly or a
competitive market.

Team of Microeconomics I Teaching Assistant | 85


d less than the level produced by either a monopoly or a
competitive market.
9 When an oligopolist individually chooses its level of production to
maximize its profits, it charges a price that is
a more than the charged by a monopoly and less than the price
charged by a competitive market.
b less than the price charged by a monopoly and more than the
price charged by a competitive market.
c more than the price charged by either a monopoly or a
competitive market.
d less than the price charged by either a monopoly or a competitive
market.
10 A situation in which oligopolists interacting with one another each choose
their best strategy given the strategies that all the other oligopolists have
chosen is known as a
a collusion solution.
b Cartel
c Nash equilibrium.
d dominant strategy.

Essay

1. Suppose that a natural monopolist was required by law to charge average


total cost. On a diagram, label the price charged and the deadweight loss
to society relative to marginal-cost pricing

Team of Microeconomics I Teaching Assistant | 86


2. Based on market research, a recording company obtains the following
information about the demand and production costs of its new CD:

Price = 1,000 – 10Q


Total Revenue = 1,000Q – 10Q2
Marginal Revenue = 1,000 – 20Q
Marginal Cost = 100 + 10Q

where Q indicates the number of copies sold and P is the price in cents.
a. Find the price and quantity that maximizes the company’s profit.
b. Find the price and quantity that would maximize social welfare.
c. Calculate the deadweight loss from monopoly.
3. Consider the relationship between monopoly pricing and price elasticity of
demand:
a. Explain why a monopolist will never produce a quantity at which
the demand curve is inelastic. (Hint: If demand is inelastic and
the firm raises its price, what happens to total revenue and total
costs?)
b. Draw a diagram for a monopolist, precisely labeling the portion
of the demand curve that is inelastic. (Hint: The answer is related
to the marginal-revenue curve.)
c. On your diagram, show the quantity and price that maximizes
total revenue
4. Shawn Mendes has just finished recording his latest CD. His record
company’s marketing department determines that the demand for the CD
is as follows:

Team of Microeconomics I Teaching Assistant | 87


The company can produce the CD with no fixed cost and a variable cost
of $5 per CD.
a. Find total revenue for quantity equal to 10,000, 20,000, and so
on. What is the marginalrevenue for each 10,000 increase in the
quantity sold?
b. What quantity of CDs would maximize profit? What would the
price be? What would the profit be?
c. If you were Shawn’s agent, what recording fee would you advise
Shawn to demand from the record company? Why?
5. Suppose that Tommy Hilfiger’s marginal cost of a jacket is a constant
$100 and the total fixed cost at one of its stores is $2,000 a day. This store
sells 20 jackets a day, which is its profit-maximizing number of jackets.
Then the stores nearby start to advertise their jackets. The Tommy
Hilfiger store now spends $2,000 a day advertising its jackets, and its
profit-maximizing number of jackets sold jumps to 50 a day.
a. What is this store’s average total cost of a jacket sold before the
advertising begins?
b. What is this store’s average total cost of a jacket sold after the
advertising begins?

Team of Microeconomics I Teaching Assistant | 88


c. Can you say what happens to the price of a Tommy Hilfiger
jacket? Why or why not?
d. Can you say what happens to Tommy’s markup? Why or why
not?
e. Can you say what happens to Tommy’s economic profit? Why or
why not?
f. How might Tommy Hilfiger use advertising as a signal? How is a
signal sent and how does it work?
g. How does having a brand name help Tommy Hilfiger to increase
its economic profit?
6. You are hired as the consultant to a monopolistically competitive firm.
The firm reports the following information about its price, marginal cost,
and average total cost. Can the firm possibly be maximizing profit? If not,
what should it do to increase profit? If the firm is profit maximizing, is the
firm in a long-run equilibrium? If not, what will happen to restore long-
run equilibrium?
a. P < MC, P > ATC
b. P > MC, P < ATC
c. P = MC, P > ATC
d. P > MC, P = ATC
7. Consider a monopolistically competitive market with N firms. Each firm’s
business opportunities are described by the following equations:
Demand: Q = 100/N − P
Marginal Revenue: MR = 100/N − 2Q
Total Cost: TC = 50 + Q2
Marginal Cost: MC = 2Q

Team of Microeconomics I Teaching Assistant | 89


a. How does N, the number of firms in the market, affect each firm’s
demand curve? Why?
b. How many units does each firm produce? (The answers to this and
the next two questions depend on N.)
c. What price does each firm charge?
d. How much profit does each firm make?
e. In the long run, how many firms will exist in this market?
8. Little Kona is a small coffee company that is considering entering a
market dominated by Big Brew. Each company’s profit depends on
whether Little Kona enters and whether Big Brew sets a high price or a
low price:

a. Does either player in this game have a dominant strategy?


b. Does your answer to part (a) help you figure out what the other
player should do? What is the Nash equilibrium? Is there only
one?
c. Big Brew threatens Little Kona by saying, “If you enter, we’re
going to set a low price, so you had better stay out.” Do you think
Little Kona should believe the threat? Why or why not?

Team of Microeconomics I Teaching Assistant | 90


d. If the two firms could collude and agree on how to split the total
profits, what outcome would they pick?
9. A case study in the chapter describes a phone conversation between the
presidents of American Airlines and Braniff Airways. Let’s analyze the
game between the two companies. Suppose that each company can charge
either a high price for tickets or a low price. If one company charges $100,
it earns low profits if the other company charges $100 also, and high
profits if the other company charges $200. On the otherhand, if the
company charges $200, it earns very low profits if the other company
charges $100, and medium profits if the other company charges $200 also.
a. Draw the decision box for this game.
b. What is the Nash equilibrium in this game? Explain.
c. Is there an outcome that would be better than the Nash
equilibrium for both airlines? How could it be achieved? Who
would lose if it were achieved?
10. The New York Times (Nov. 30, 1993) reported that “the inability of OPEC
to agree last week to cut production has sent the oil market into turmoil . .
. [leading to] the lowest price for domestic crude oil since June 1990.”
a. Why were the members of OPEC trying to agree to cut
production?
b. Why do you suppose OPEC was unable to agree on cutting
production? Why did the oil market go into “turmoil” as a result?
c. The newspaper also noted OPEC’s view “that producing nations
outside the organization, like Norway and Britain, should do their
share and cut production.” What does the phrase “do their share”
suggest about OPEC’s desired relationship with Norway and
Britain?

Team of Microeconomics I Teaching Assistant | 91


CHAPTER 9
GENERAL EQUILIBRIUM & MARKET EFFICIENCY

• Partial equilibrium analysis is the process of examining the equilibrium


conditions in individual markets and for households and firms separately.

• General equilibrium is the condition that exists when all markets in an


economy are in simultaneous equilibrium.

• Efficiency is the condition in which the economy is producing what


people want at least possible cost.

• Pareto efficiency or Pareto optimality is a condition in which no change


is possible that will make some members of society better off without
making some other members of society worse off.

• The three basic questions discussed previously included:


o What gets produced? What determines the final mix of output?
o How is it produced? How do capital, labor, and land get divided
up among firms? In other words, what is the allocation of
resources among producers?

o Who gets what is produced? What determines which households


get how much? What is the distribution of output among
consuming households?

• Market failure occurs when resources are misallocated, or allocated


inefficiently. The result is waste or lost value.

• There are four important sources of market failure:


1. imperfect market structure, or noncompetitive behavior,
2. the existence of public goods,
3. the presence of external costs and benefits, and

Team of Microeconomics I Teaching Assistant | 92


4. imperfect information.

• Imperfect condition is an industry in which single firms have some


control over price and competition. Imperfectly competitive industries
give rise to an inefficient allocation of resources.

• Monopoly is an industry composed of only one firm that produces a


product for which there are no close substitutes and in which significant
barriers exist to prevent new firms from entering the industry.

• Public goods, or social goods are goods or services that bestow collective
benefits on members of society. Generally, no one can be excluded from
enjoying their benefits. The classic example is national defense.

• Private goods are products produced by firms for sale to individual


households.

• Externality is cost or benefit resulting from some activity or transaction


that is imposed or bestowed on parties outside the activity or transaction.

• Imperfect information is the absence of full knowledge concerning


product characteristics, available prices, and so forth.

Team of Microeconomics I Teaching Assistant | 93


CHAPTER 9
GENERAL EQUILIBRIUM & MARKET EFFICIENCY

Fill in the Blank

1. If all assumptions about perfectly competitive markets hold, then the


allocation of resources is _____.

2. A condition in which no change is possible that will make some members


of society better off without making some other member of society worse
off is _____.

3. Oligopolistic industries are made up of a ____ number of firms, while


monopolistically competitive industries are made up of a ____ number of
firms.

4. Product differentiation is how _________ industries acquire pricesetting


power.

5. _______ produce less output and charge higher price than perfect
competition market.

6. Price ______ marginal cost is the condition where the perfect competition
market will produce output in short run and long run.

7. _____ is the best estimation of the utility value derived from consumption.

8. The effect of technological changes on costs, firm entry, and market price
in a given industry can be considered ____________.

9. The condition that exists when all markets in an economy are in


simultaneous equilibrium is called ________.

Team of Microeconomics I Teaching Assistant | 94


10. Output and input markets are connected. If product demand increases,
marginal revenue product will _____, and the _____ curve will shift to the
right.

True or False

1. Perhaps the best feature of the market system is that it always takes into
consideration all of the costs and benefits of decisions made by
households and business firms.

2. In imperfectly competitive industries, output and price are higher than


they would be under perfect competition.

3. Sometimes it is the government that prevents the entry of firms into a


market, thereby stifling competition and preserving the monopoly status
of a business firm.

4. If price is greater than marginal cost, the amount of output produced


should increase.

5. The allocation of resources among firms is efficient only if all firms pay
the same prices for inputs.

6. If all the assumptions of perfect competition hold, the result is a Pareto


optimal allocation of resources.

7. A pareto optimal system is a system in which changes in the allocation of


resources make some people better off without making others worse off.

8. Market failure occurs when resources are misallocated and the result is
lost value.

9. When an economic system produces what people want and does so at the
least possible cost, the economy has achieved effectiveness.

Team of Microeconomics I Teaching Assistant | 95


10. If the market is a perfect competitive market, the shift in the market
supply curve to the right, ceteris paribus, is caused by higher cost,
potential for loss, and consequently exit of new firm.

Multiple Choice

1. Which of the following is true about imperfect information?


a. Markets work efficiently only if some consumers and producers have
imperfect information

b. The absence of full information can lead to transactions that are


ultimately disadvantageous

c. Misinformation is difficult to correct in most cases


d. Practically every industry has found a solution to the problem of
misinformation

2. Which of the following sources of inefficiency deals with the presence of


costs and benefits not accounted for in prices?

a. Imperfect information
b. Imperfect markets
c. Public goods
d. Externalities

3. Which of the following should we expect in a completely laissez-faire


market system?

Team of Microeconomics I Teaching Assistant | 96


a. We can expect private producers to produce all the goods and
services that society wants, thus there would be no need for public
goods

b. The private market would not produce some of the goods people
want, thus we would have to rely on the government to produce
some goods
c. All the goods that society wants would be public goods, thus there
would be no need for a private sector

d. The private and public sectors would cooperate with each other to
provide the goods that society wants most

4. Which of the following are sources of market failure?


a. Public goods
b. Imperfect markets
c. Imperfect information
d. All of the above

5. In order for output to be efficiently distributed among households in free


and open markets, which of the following conditions is/are necessary?

a. The quantities households purchase of a given good must be the same

b. The prices households pay for a given good must be the same
c. The tastes and preferences of households for a given good must be the
same

d. None of the above. Prices, quantities purchased, and tastes and


preferences can all be different

Team of Microeconomics I Teaching Assistant | 97


6. The allocation of resources in perfectly competitive, free, and open markets
id efficient when ……..

a. No single firm can get more value out of a factor of production than
that factor’s current market price

b. All firms have access to the same factor markets and the same set of
prices in input markets
c. All firms maximize profit
d. All of the above

7. An efficient economic system is a system in which ……..


a. Households have perfect information on product quality and all prices
available

b. Firms have perfect knowledge of technologies and input prices


c. Firms produce the right type and amount of output, or the output that
people want most, at the least possible cost.

d. There are both internal and external costs

8. Economists use specific criteria to judge the performance of economic


systems and to evaluate alternative economic policies.

These criteria are ……..


a. Allocative efficiency and pareto optimality
b. Efficiency, equity, stability, and growth
c. Maximum well-being for society, and/ornthe potential for market
failure

d. Perfect competition and pareto efficiency

Team of Microeconomics I Teaching Assistant | 98


9. Refer to the figure below. After the change in market demand, which of the
following is likely to occur?

a. The supply curve will also shift to the right


b. The supply curve will shift to the left
c. A further change in market demand, but no change in market supply

d. There is insufficient information to predict the next move in this


market

10. When a significant technological change affects one industry, which of the
following is likely to be affected?

a. Costs, output, and prices are likely to be affected in the industry but
probably not in other industries

b. The change will affect many markets, except the labour or capital
markets

c. The change could affect many markets, including the markets for
labour and capital.

d. The supply side of the market is affected, or production, but not the
demand side, or consumption

Team of Microeconomics I Teaching Assistant | 99


Essay

1. How do we measure the value or marginal benefit of a good or service?

2. What is consumer surplus and producer surplus? Give example of how these
are measured!

3. What is deadweight loss and under what condition does it occur?


4. What are the obstacles to achieve an efficient allocation of resources in the
market economy?
5. The table gives the supply schedules for jet-ski rides by three owners:
Rick, Sam, and Tom, the only suppliers of jet-ski rides.
Price (dollars per ride) Quantity Supplied (rides per day)

Rick Sam Tom

10.00 0 0 0

12.50 5 0 0

15.00 10 5 0

17.50 15 10 5 20.00 20 15 10

a) What is each owner’s minimum supply-price of 10 rides a day?


b) Which owner has the largest producer surplus when the price of a
ride is $17.50? Explain!

c) What is the marginal social cost of producing 45 rides a day?


6. How do we know if a market is producing the socially optimal quantity of
each output?

Team of Microeconomics I Teaching Assistant | 100


7. Why does the monopoly cause the market failure in the economy?
8. Explain the characteristics of public goods!
9. Why do we study the efficiency properties of perfect competition?
10. How can one tell if an allocation is pareto efficient?

Team of Microeconomics I Teaching Assistant | 101


Team of Microeconomics I Teaching Assistant | 102

Вам также может понравиться